0

Moin zusammen, habe folgende Aufgabe gelöst:



Meine Lösung:

Definiere $h(x) = f(x) - g(x)$. $h(x)$ ist stetig weil $f(x),g(x)$ stetig sind.
$g(x)$ ist beschränkt d.h. $-C \leq g(x) \leq C$ wobei $C>0$.
Weil $f$ surjektiv ist $\exists x_1,x_2 : f(x_1) = -C\ \land f(x_2)=C$  (o.B.d.A $x_1 \leq x_2$).
Auswerten von $h$ an $x_1$ und $x_2$:
$x_1$: $h(x_1) = f(x_1) - g(x_1)= -C - g(x_1) \leq -C+C=0$ (weil $-g(x_1) \leq C$ )
$\implies h(x_1) \leq 0$ 
$x_2$: $h(x_2)=f(x_2)-g(x_2)=C-g(x_2) \geq C-C=0$ (weil $-g(x_2) \geq C$ )
$\implies h(x_2) \geq 0$ 

Daraus folgt, dass $h(x_1)$ und $h(x_2)$ unterschiedliche Vorzeichen haben oder beide $0$ sind.
Mit dem ZWS folgt jetzt 
$\exists y_0\in [h(x_1),h(x_2)]$ mit $y_0=0=h(x_0)$ sodass $x_0\in [x_1,x_2]$.
$\implies h(x_0) = 0 = f(x_0) - g(x_0) \implies f(x_0) = g(x_0)$
$\blacksquare$

Leider habe ich keine Lösungen dazu. Stimmt der Lösungsweg ansatzweise?

 

 

Diese Frage melden
gefragt

Student, Punkte: 93

 
Kommentar schreiben
1 Antwort
0

Das Schielen nach einer sogen. "Musterlösung" ist anscheinend derart eingebrannt, dass man sich ungern davon verabschiedet. Naja, kommt hoffentlich später noch.

Dein Beweis ist vollkommen in Ordnung.
Man kann ihn noch etwas rundschleifen:
Wähle $C$ so, dass $-C<g(x)<C$, dann haben $h(x_1)$ und $h(x_2)$ wirklich verschiedene Vorzeichen (der "Nullfall" entfällt).
Also hat $h$ eine Nullstelle in $[x_1,x_2]$ (es gibt kein Grund hier eine neue Größe $y_0$ einzuführen, die sowieso 0 ist).

Diese Antwort melden
geantwortet

Lehrer/Professor, Punkte: 39.01K

 

Hi mikn, danke für deine Antwort.
Hoffe ich auch :-)
Das $C$ ist in der Aufgabenstellung so gegeben. Aber statt $C$ so umzuändern, oder eine andere Variabel einzuführen, könnte ich auch $x_1,x_2$ so wählen, dass ich für $f(x_1)=-2C$ und für $f(x_2)=2C$ habe.
Dann ergibt sich für $x_1$: $h(x_1) \leq-2C+C<0$ und für $x_2$: $h(x_2) \geq2C-C>0$.
Würde auch passen, oder?
  ─   aequus formidus 09.08.2023 um 22:18

Achja, stimmt, $C$ war gegeben. Ja, $2C$ würde es tun, oder $C+1$, oder "o.B.d.A. $-C\lt g(x)\lt C$" und weiter wie oben.   ─   mikn 09.08.2023 um 22:24

Kommentar schreiben